Sei sulla pagina 1di 58

Case

Studies
1. A plane with 200 passengers on board has been hijacked by terrorists. The
plane is grounded in a country in which a number of the hijackers associates
have been imprisoned for acts of murder committed ruthlessly in that country
and in others. The hijackers demand the release of all these prisoners, and state
that they will exterminate all the passengers if this is not done. To show they
mean business, they shoot the youngest passenger, a child of 6, and drop her
body onto the concrete. What should the authorities do? The hijackers are
known to have enough weapons to blow up the plane; 200 innocent people could
die; but by acceding to these demands, the authorities could be giving the allclear to similar potential hijackers throughout the world. On what principle
should the solution be determined?
Answer
Ethical Issues involved
Whether to listen to the Hijacker
Passenger Safety
Release or not to release Prisoners.
To risk life by lying to the Hijackers
Options Before the government

Try and fight the hijackers

Obey the hijackers

Try and play smart by fooling them

Disobey the hijackers outrightly

Consequences of the Actions

The Government will take the decision to let the prisoners free will wait for
the last possible moment, in order to make sure that theres no third option.

Its not certain that the hijacked plane will actually be blown.

These things are never certain. However, lets assume a case in which its very
likely whatever the hijackers say is true. Catastrophic loss of life will occur if
we dont listen to them.

The people in the plane therefore look like theyre doomed whatever decision
is made that they will be left to die.

Another reasoning would favour the first option, because not releasing
prisoners means intentionally killing the passengers and thereby treating them
as instruments in an operation to keep the prisoners

Theres an issue of human dignity here. People shouldnt be instrumentalized.

So we have two options: Release the Prisoners and Save the people or just
shoot the plane.

The former is in the best for everyone, for the government should value the
people inside the Plane than Prisoners and make sure they are safely rescued.

2. You are a project engineer and assisting your project director along with his
brother in law, appointed on a contract basis, in developing a presentation that
would get your cash strapped PSU organization significant funding for a new
power generation project from central government as well as private investors.
Now, when it comes to actual presentation before government officials and
private investors, director assigns his brother in law to present the plan. As
expected, this project plan gets approved with huge appreciation of the
suggestions you had added to the plan. On this, much to your surprise, project
director passes all the credit to his brother in law, for which he receives open
adulation. Later, when you question your director regarding this unethical
behavior, he tries to convince you that you, being a very bright engineer, would
come across many other opportunities to prove your caliber but this small favor
would get his brother in law permanent appointment in this PSU. What should
be your future course of action?
1. You need to raise the issue in the internal grievance cell of your
organization
2. You need to raise the issue because of following reasons-

Selection and promotion based on a criterion other than merit will promote
nepotism in organization. Inaction on your part could further enhance such
situation in future also.

Moreover efficiency of organization depends on efficient and able members,


thus the same needs to be be incorporated in the organization.

Further utilization of resources depends on efficient employees.

If such an issue is disclosed in future among other employees, it may result in


their demotivation.

3. A convicted mobster decides to make a charitable contribution. He offers


more than $1 million to a hospital to build a childrens wing. He will make the
contribution if the new pavilion is named after him. The hospital board accepts
the gift, with that stipulation. Do you think the hospital was right in accepting
the gift?
Answer
Adhering to ethical standards in fundraising is especially important because the
success of an organizations mission rests on trust: the trust of clients, volunteers,
donors, and the community served.
The Problem: Doing Good With Something Obtained Immorally
The conflict in this story stems from the strain between hospital's need for the money
and the money's being tainted and coming with a controversial string attached.
Some questions to ask yourself

How large was the majority in favor of this decision?

Were other potential benefactors approached and what were their responses?

Was the decision forced because without this money there would be no new
wing?

Was the board approached or did it solicit this donation?

How do you respond to the criticism that the money is "ill-gotten" gains? How
do you think the public will react.

Apart from this the facts are also needs to be filtered through some general principles
regarding the uses and abuses of money, and the relationship between means and
ends. It also presents a problem of proportionality. What I mean is how much bad is
done in the cause of doing how much good.
Making the Best of a Bad Situation
In regard to the case presented here, there is no question about the origins of the
money. The donor is a convicted mobster and the money is tainted. Now he wants to
take his money and wash it by putting it to a socially acceptable purpose. The
temptation to take it is great.
One approach could be
If this mobster wants to put his money into helping children, then let him. Take the
money, put his name on the hospital and use the money to treat sick children, even
save their lives in many instances.
But you need to consider the effect of such action that is, will using something bad
encourage others to do bad things?
4. You are posted as a DM in a district. You come to know that a newly
appointed BDO is making commissions from developmental activities being
carried out. Before taking action, you plan to do some background check and
come to know that this BDO is from a very humble background and his relations
are dependent on him. Moreover, he is funding the education of their children.
You also come to know that this BDO is pro-poor and has implemented all BPL
schemes with efficiency, wherever posted. Keeping these facts in mind, what
would be your course of action?
Answer
As a DM its your responsibility to maintain transparency and service orientation in
administration of district.
You need to hold a in-house inquiry to check the authenticity of information.
If the complaint holds true then it becomes your responsibility to hold the BDO
accountable for his actions.
In a public organization, means should justify ends; unfair means could never bring
welfare.

Moreover in future if through RTI or through Media such information is disclosed or


comes in public, it will lead to distrust among public towards action of the
administration.
5. BAASHA, a data analyst for a major casino, is working after normal business
hours to finish an important project. He realizes that he is missing data that had
been sent to his coworker Antony. BAASHA had inadvertently observed Antony
typing his password several days ago and decides to log into Antonys computer
and resend the data to himself. Upon doing so, BAASHA sees an open email
regarding gambling bets Antony placed over the last several days with a local
sports book. All employees of the casino are forbidden to engage in gambling
activities to avoid any hint of conflict of interest.
BAASHA knows he should report this but would have to admit to violating the
companys information technology regulations by logging into Antonys
computer. If he warns Antony to stop his betting, he would also have to reveal
the source of his information. What should BAASHA do in this situation?
Answer
In the given case study, BAASHA would have to choose between saving himself
from trouble and knowing that he is letting his coworker do something wrong, or else
landing up in trouble to correct a wrong that his coworker is committing.
One option is that he does not tell on Antony and himself. Another option is that he
would tell on themselves to get Antony and himself in trouble. Yet another possibility
is that he may tell on Antony, but try to clear himself.
If I were in this situation, I would keep to myself the information I found about
Antony and inform Antony that I accidentally observed him typing in his password.
Then he would probably change it. Since it was morally wrong of me to log into
Antony's computer; therefore information I found by doing something wrong should
not be used to call someone else out on their wrong. If I later overheard Antony
talking about his gambling or found out a legitimate way, I would report it then.
6. The mood at Vidya Mandir High School is tense with anticipation. For the
first time in many years, their basketball team has made it to the state semifinals.
The community is excited too, and everyone is making plans to attend the big
event next Saturday night. Vijay, the varsity coach, has been waiting for years to
field such a team. Speed, teamwork, balance: they've got it all. Only one more
week to practice, he tells his team, and not a rule can be broken. Everyone must
be at practice each night at the regularly scheduled time: No Exceptions. Ajith
and Mohan are two of the team's starters. From their perspective, they're
indispensable to the team, the guys who will bring victory to Vidya Mandir.

They decidewhy, no one will ever knowto show up an hour late to the next
day's practice.
Vijay is furious. They have deliberately disobeyed his orders. The rule says they
should be suspended for one full week. If he follows the rule, Ajith and Mohan
will not play in the semifinals. But the whole team is depending on them. What
should he do?
Answer
Everybody waited for so many years to get a right team to win the Championship and
both Ajith and Mohan are key players. The ultimate aim is to win the championship
and without them it would be difficult. But if Vijay doesnt punish them there are
chances that other students might do similar things. Then the law he made wont be
valid.
One option is that the coach should ban them; the rules have to be the same for all
children. Fair play and sticking to the rules is the most important thing, it doesnt
matter if the team wins or loses. The boys knew what they were doing, they shouldnt
have taken a risk like that.
Another option is that the coach should let them play in the match and they should be
suspended for a week after the game is over. Its not fair that the other team members
have to go without the best players, they wont win if the boys are not allowed to
play. The dreams will be over.
Yet another option is that maybe the coach could just give them a warning. The coach
should let them play but he could warn them and put them on probation, if they break
the rules again they will get suspended from the school, but if they dont they will
have learnt their lesson.
I think coach should ban them for sure but that doesnt mean that the team wont win.
It doesnt matter how good or important you are, the same rules should apply to all.
There is no evidence in the story that these rules were any less fair for these two boys.
They are part of the team and so they have entered in a relationship of trust and
community with the other members on the team. Unless there was a good reason for
them to be late, they have let down the other members of their team community and
so I think they deserve to serve the agreed suspension.
7. You head the public relations department of the states largest bank. The
department is responsible for putting together a quality service recognition
program. Your banks public relations agency is designing the advertising
specialty components for the program targeting the banks 10,000 employees.
Your spouse owns X Promotions, the largest advertising specialty firm in the

state. The company offers the best prices for large orders. X Promotions has
supplied products for a number of other accounts of the public relations firm.
This is the first time, however, that the public relations firm has used X
Promotions for a bank project. The public relations firm does not know that
your spouse owns X Promotions. You have not suggested the use of X
Promotions. The public relations firm has made its recommendations to you,
including using X Promotions as the vendor for the quality service recognition
program. What should be your next course of action - with the public relations
agency, your management team and your spouse?
Answer
Following questions need to be asked:

Do I inform my banks management about the potential conflict of interest?

Should I ask the public relations firm to select another vendor?

Should I let the public relations firm pick the vendor? After all, I didnt force
them to pick my spouses company. X Promotions did have the best price.

Building trust with the employeesis key in this case. Even though the use of your
spouses company would most likely result in a good and best price for the bank,
letting the public relations agency use the company to supply the components for the
quality service program would present a definite conflict of interest.
You should immediately notify the public relations agency not to use your spouses
company as a vendor for this and all future bank programs. You should explain that
employees might perceive a conflict of interest, and that the perception would damage
mutual understanding and credibility. The lack of trust by employees might be
extended to the entire management of the bank. The loss to your spouses company
might be substantial, but you need to put the interests of your employer and its
employees before your personal interests. Even if you revealed the conflict to your
banks management team, and they approved the supplier, employees might still
perceive that you personally benefited from the bank using your spouses company as
a supplier. Avoiding the conflict would guarantee continued mutual respect between
you and employees.
8. In an unexpected turn of events, you find a bag full of money. You somehow
come to know that this is black money, earned through unethical acts. Due to all
pervasive corruption, you are facing a dilemma on handing over the money to
the administration. You have a very close and honest friend who runs an

orphanage. The orphanage is in a very petty situation due lack of finance. To


whom would you give the money and why?
Answer
The dilemma is the case is that the black money, if given to the administration, can be
misused. At the same time if it is given to orphanage than it can be best utilized.
However, there are certain negative consequence of this action too:

It cannot be established as a rule. Because if everybody starts adopting such


practices there shall be lawlessness.

It is my perception that my friend will use fund for orphanage, but he may also
end up misusing it.

If I may not hand over the money to the administration, it may lead to a habit
of not abiding by the law.

It is public money and the government/administration is the final arbitrator for


its planned utilization.

If money is not handed over to the administration and it comes to their


knowledge later, then there could be legal repercussions for me.

Thus, I shall find out that what is the mechanism to deposit the money to the
administration. I shall try my level best that money is utilized for its intended
purpose, in the interest of the public exchequer.

9. You are the public relations professional for a housing developer. Your
companys next project is a multi-family housing complex for middle-income
families. While gathering information related to the project, you find that land
for the housing complex was the site of a landfill. An EPA report shows very low
levels of contaminants that are not life threatening. You discuss this information
with your supervising boss, who is not a public relations professional, and
request him to recommend ways to explain the landfill history in promotional
materials. Your boss tells you that the landfill information is not to be included
in the materials. He does not want this issue to be proactively discussed. What
should you do in this situation?
Answer
Ethical principles to guide the decision making process.

Enhancing the Profession -- To build respect and credibility with the public for the
profession of public relations. This principle requires that a member decline
representation of clients or organizations that urge or require actions contrary to this
Code. Additionally, unethical behavior would be to declare the product (in this case,
land) safe without disclosing the EPA report.
Disclosure of Information To build trust with the public by revealing all information
needed for responsible decision-making. This principle requires that a member be
honest and accurate in all communications.
Course of action
There is a potential for conflict between loyalty to the employer and the publics right
to know. However, the value of independence helps resolve this conflict. You must
convince your boss that it is in the companys best interest to acknowledge the history
of the land. It is an opportunity to explain all efforts to ensure the safety of the land
being developed and future monitoring. The reputation of the company will be
enhanced by proactive communications.
If the EPA report becomes public under Freedom of Information Act, the failure to
voluntarily disclose this information could backfire against the developer and result in
serious economic losses.
Your reputation as a public relations practitioner depends on your skills to persuade
your employer to act in the best interest of the public.
10. You and Archana are the respective in-charge of two
government organization, with both working under the General
Your organization contracts out work to two private sector
Services and Vijaya Services. Both of them are fully competent
good work.

subunits in a
Manager Bala.
firms: Shreya
and have done

Both you and Archana have encouraged Bala to spread the work between these
two contractors equitably to create some beneficial competition. But over the last
3 years, only Shreya Services has received the larger contracts, which transcends
the level of your subunits and thus are out of your purview. Bala insists that
Shreya Services has done fine work and there is no need to change. He talks
about continuity and proven performance, and argues that stability serves the
good of the department by reducing transaction costs and thereby increasing
efficiency.
Archana has argued for including Vijaya Services in the major contracts, not
only because it does good work but because it is a smaller firm that needs
government contracts to get well established. You believe that a combination of

equally excellent performance and compensatory justice for smaller firms that
are struggling to compete, make Vijaya Services even more deserving of a share
in the big contracts. However, Bala is not willing to budge from his seemingly
unreasonable position.
Vijaya Services being a firm held by the minority community, you and Archana
discuss the possibility that Bala may be biased in consistently rejecting Vijaya
Services out of hand for the bigger contracts. The two of you try to find out an
alternative explanation for Balas conduct, but it is difficult not to interpret his
actions as being motivated on communal grounds.
Design an approach to address this serious problem of apparent inequity and
perhaps communal bias. Keep in mind that this is not just an isolated incident; it
represents a pattern of practice with potentially serious consequences for the
organization and the contractors and implications for cultural norms of the
organization. List the steps that you would take to address the problem.
Answer
The approach to deal with this serious problem of apparent inequity and communal
bias can include the following steps:
1. Define carefully and systematically the ethical problem we are facing. Is it
communal bias or unfair treatment for some other reason? Because the act of simply
confronting the boss with a charge of communal bias is not likely to elicit an honest
response if he is biased in that way, we will need to examine other evidence of
communal bias in his conduct. Has he willingly hired, encouraged, and promoted
people of minority communities? Is there any other evidence of bias in the incidences
he tells or the way he favours people in the organization? If there is no other evidence
of communal bias, we may have no reason to believe it exists in this case.
If there are no good reasons to believe communalism is at work in the selection of
contractors, we will need to try to determine why he is consistently favouring Shreya
Services. Is he receiving kickbacks from Shreya? Does he have an old friend or
relative who is in a key position in that firm? Or is it something else? Does he just
feel comfortable staying with a firm he knows and trusts? In order to determine what
is influencing Balas actions, we may need to talk discreetly with others in the office
who have been in the organization for a long time about his relationship to Shreya
Services. We may need to carefully talk with people we know at Shreya and Vijaya
Services about this by raising indirect questions. Eventually we may need to confront
the boss with our concern and ask him why he does not see the equity problem in
excluding Vijaya Services from the large contracts that he controls. We should always
be open to the possibility that he really believes what he espouses: that continuity and
stability best serve the good of the organization.

With a better focus on the nature of the ethical problem, we try to identify what we
think is the best course of action to address the problem. In this case, the facts show
that Bala repeatedly grants bigger contracts to Shreya and refuses to consider using
Vijaya Services, even though it performs well on smaller contracts. The principle at
stake is equity and the possibility of Bala being communally biased in his
administrative alternatives. Is there a mechanism within our organization to report our
concerns? Is there a proper chain of events for documenting and reporting such
things? Is our first step to confront Bala directly? For each possible solution, we must
identify the probable consequences both negative and positive. Ideally the positive
result would be for some of the larger contracts to be awarded to Vijaya. However, we
must also consider an end result that is negative. For example, we may be labeled as
disloyal or insubordinate if we report on Balas actions. Or our relationship with Bala
may suffer, impeding our ability to do well in our job and succeed in our agency. The
next step is to find the most fitting alternative, knowing that we will likely have to
defend our actions and perhaps take some blows to our self-image or our reputation
among our coworkers. Once this is done, we can select our alternative
3. With a course of action identified, we turn to a consideration of the organizational
factors that may encourage or impede what we are proposing to do. Is the
organization so hierarchical that it is difficult to deal with unethical decisions made
above our level? Does the culture of the organization support arguing with the boss
when that seems to be required? Can one go around the boss to the next person up the
chain of command without destroying ones career? Are there dissent channels, either
formally established to deal with problems of professional conscience or set up with
other purposes, that can be used to express concerns about perceived misconduct (for
example, a human resource department that might function as such a channel)?
4. Having developed an analysis of the organizational structure and culture, the next
step is to consider intervention strategies we might employ to make the organization
more supportive of the kind of conduct we have decided is consistent with the ethics
of the public administrative role. Much depends on how we have defined the ethical
problem. If it is a matter of communalism, we might try to get communal sensitivity
training sessions established in which the boss would participate. If it is not
something that blatant, we might consider pushing for the creation of a contract
review committee within the organization to periodically look at all the contracts
awarded during a given period, examine the criteria, and assess the fairness of the
distribution of awards. Or we might decide the best way to handle problems of equity
in contracting is to avoid having any single person in charge of approvals and instead
to rely on collective decision making among top management. Beyond measures of
this kind, we might decide to recommend a thorough organizational development
project to restructure the organization, clarify its values, institute dissent channels,
and create accountability arrangements designed to check favouritism or tendencies to
just do the same old thing because it is comfortable to do so.

11. Mr. X is the engineering manager for the County Road Commission (CRC),
with the primary responsibility for county road safety. Along a given stretch of
the road, many people have died over the course of last 5 years by crashing into
roadside trees. Many other accidents have also occurred. Two lawsuits had been
filed regarding the unsafe segment of the road, but were dismissed because the
drivers were exceeding the 45 mph speed limit. Mr. X recommends that the road
be widened, cutting down a large number of trees in the process.
This leads to a protest by environmental groups and they file a petition to save
the trees; and instead educate people about following the road safety rules like
speed limit etc. The public sentiment is divided on the issue with no sight of a
wide consensus. Discuss how Mr. X should proceed at this point.
Answer
In discussion of this situation, one can try responding to the following:
What do societal values require in this case?
What do professional ethical standards require in this case?
Societal values:
A consideration of a variety of societal values would be relevant in this situation. For
instance, since society values human life, so this would favor widening the road and
sacrificing the trees.
However, society also values environmental preservation, in which case a solution
that does not kill the trees should be found
Professional standards demand that efforts be made in improving the public
knowledge regarding the issue -this indicates that whatever decision is made, an effort
should be made to educate the public holistically on all the related issues.
Recommended action for Mr. X:
(1) First, hold a public meeting to inform the public about the decision you may have
arrived at. However, it should be noted that presently there may not be wide ranging
consensus on the issue. This is because the people who have presented the petition
may not constitute the entire population of the town. There may be a silent majority
who hold a different view.

(2) One way to resolve the issue is to widen the road and remove trees from the rightof-way for optimal safety.
(3) But the county should relocate the displaced trees to other public property (a park,
etc).
12. You have just been appointed as a director of the city municipal corporation.
A joint director, much senior to you in age and about to retire in six months, is
passionately working on a very important urban planning project, successful
completion of which would earn him a lasting reputation for the rest of his
retired life. A new lady Civil Engineer has joined the corporation, having
background from a leading premier institution in this field. This has made the
Joint Director (JD) so insecure that he constantly fears that she would take all
the credit. He has adopted a passive aggressive behavior towards her and is
disrespectful in his communication to her. The lady engineer feels embarrassed
when the Joint Director corrects her in front of other employees, raises his voice
when speaking to her and leaves no chance to humiliate her. She may also be
intimidated by him since he has had a long tenure in the office, has directly
related experience in the functional area that she is working in, and was also
favored by the previous Director. She may be feeling that she has no recourse in
the matter. You are well aware of her outstanding academic and career record in
previous organizations but fear this one sided ego clash from the side of the Joint
Director would seriously compromise her much needed contribution in this
important project and also, her emotional well-being. Latest, you come to know
that she is planning to resign. How will you handle this situation?
Answer
Help the Joint Director understand that

She will not be allowed to walk away with the credit of his work and assure
him that as the Director you will ensure that he will be given due credit for his
work as the Joint Director heading the project given to him.

If because of him, she resigns and in the process puts a complaint, it might
become a blot on his career.

He should rather help create a good team of subordinates, even after he leaves,
who will hold him in high esteem once he leaves and will be a good
contribution from his side to the Corporation.

The lady engineer has an outstanding academic and career record in previous
organizations and good potential and he should help her realize it and play a

good Mentors role as he is her senior and has directly related experience in
the functional area that she is working in. this is also his professional
obligation in some sense.

That his behavior and attitude towards her is non-professional and


unwarranted.

Make him realize how one sided ego clash from his side would seriously
compromise her much needed contribution in this important project which is
also his last project and her contribution instead could enhance the overall
outcome getting him the final credit as, after all he is in charge of the Project.

Such unprofessional attitude will also impact his efficiency and divert his
focus towards non- productive aspects.

Help the new lady Civil Engineer understand

How and why the Joint Director is passionately involved

Why and how she should get the JD into confidence by proving to be a
dedicated and supportive subordinate. By showing the professional respect to
her senior and by being a willing-to-learn subordinate she will satisfy the JDs
ego as a senior. This way she will get lots of opportunities to learn from the
JDs experience.

That she can discuss the related dimensions one to one in advance, as a good
and helping subordinate, to avoid any in-public awkward situations.

That she can show better professionalism by being assertive when required but
not aggressive in reply.

That as the present Director, you will support her find her footing in the
organization, if she shows patience and professionalism in times of stress and
even after the JD retires.

That you are impressed by her academic and career record and it will help her
go a long way in the Corporation and that in career one needs to handle
different types of stress.

That you will always help her in overcoming any troublesome situation and
she should not think of resigning, as it will not reflect well in her career record
for quitting too soon.

Further make them interact more in your presence where you could balance and
control the situation as the senior most official by position, ensuring the desired
outcome for the Corporation and publicly also showing that you will take a just stand,
encouraging both for their respective efforts.
13. A state-of-the-art technology product is to be launched by a leading company
on a widely advertised date, simultaneously at different locations in the country,
for the first time. There is fantastic customer response and heavy bookings for
the product
A big event is planned in Trichy for the launch to which a large number of
dignitaries, customers and media have been invited, thus ensuring extensive
press coverage. The managers career hinges on the success of the event and the
launch.
Three days before the scheduled launch date, the managers deputy tells him
that the trucks transporting the product have been detained at the octroi post
outside Trichy, ostensibly for want of some documents, and the octroi inspector
is demanding a bribe for clearance of the consignment.
Should we pay the bribe? the deputy manager asks the manager. What should
the manager do? Discuss the course of action he can take. Keep in mind, that this
is not an isolated incident. These kinds of things happen a lot in the country
where a bribe is demanded for quick solutions. Discuss an approach, which
can be followed in such situations.
Answer
The deputy manager may face a number of ethical dilemmas that may complicate his
decision.

Should the deputy manager do anything, legal or illegal, in order to ensure a


successful launch...? Or should he act within bounds of the law and ethical
propriety...?

Why has his deputy manager passed the ball in the managers court...? Such
situations must have arisen before. Is it a trap to test the managers honesty...?

The manager has many ethical obligations towards several parties.

He has an obligation to obey the laws of the land but as an employee he also
owes a degree of obedience to his superiors and an obligation to ensure the
companys success. This conflicting two- fold obligation comprises not only

business and commercial success but also includes his duty to guard his
companys reputation, protect its interests and see that it doesnt fall foul of
the law.

Finally, the manager has a duty towards himself not to compArchanase his
own personal conscience.

A holistic view of the ethical perspectives of concerned stakeholders involved in the


ethical dilemma will provide a solution. In the ethical situation analyzed here, in this
case study, it clearly suggests that in the long-term interest of the stakeholders
involved, the manager should:
1. Decline to pay the bribe.
2. Apprise the top management of his decision.
3. Use the three days time available and try to resolve the issue in a proper manner,
with the help of the top management, intervention at higher levels and threat of
counter-exposure if necessary.
4. Take customers into confidence to cater for the worst case scenario.
5. Ask the top management to promulgate a code of ethics, which clearly prohibits all
types of bribes and illegal payments.
Business is a cooperative activity whose very existence requires ethical behaviour, as
any unethical behaviour on the part of any stakeholder is detrimental to business
interests. Business cannot strive without ethics, so it is in the best interest of business
to promote ethical behaviour among all its stakeholders as well as within its larger
society. When employees believe an organization is ethical, they are more willing to
contribute to the organizations interests, as they see managers leadership as
legitimate and readily follow what their managers and supervisors tell them to do.
Thus, ethics is sine qua non for any business.
14. A junior member of staff has just returned to work after taking special leave
to care for her elderly mother. For financial reasons she needs to work full-time.
She has been having difficulties arranging proper care for her mother, which has
led her to miss important team meetings (usually taking place at the beginning of
each day) and to leave the office early. She is very competent in her work but her
absence is putting pressure on her as well as her overworked colleagues. You
being her manager are aware that the flow of work is coming under pressure due
to this. One of her male colleagues is beginning to make comments such as a
womans place is in the home, and is undermining her at every opportunity,
putting her under even greater stress. How will you deal with the situation?

Answer
Fundamental Principles
1. Integrity: You need to be fair to all those involved and act in a straightforward
manner.
2. Confidentiality: You have a duty of maintaining confidentiality to the staff
involved.
3. Professional behaviour: How should you proceed so as not to discredit yourself,
your profession or the practice for which you work?
Consider not just who you should involve but also why and when. Do you have
access to appropriate staff in HR? Can you consult someone in the office in whom
you can confide?
Possible Action
Check the relevant facts: Clarify staff procedures with the senior HR manager. Take
legal advice if required.

Discuss the matter with the staff member: Suggest a more flexible approach to
team meetings do these always have to be in the morning? Working from
home may be an option for the junior staff member, if possible.

Remind the male member of staff about proper conduct and how such
behaviour may amount to harassment and he might have to face legal
problems along with the chance of spoiling companys reputation.

Throughout, you must be seen to be acting fairly: both towards the junior
member of staff, who is responsible for her mothers care, and towards other
members of staff.

15. Consider that you are working in an organization. The Chairman of the
organization has made it a practice even in good times to have his senior
executives fire ten percent of their subordinates once every three years, and
those at lower levels to do the same. Its all about performance. Some think that
its cruel or brutal to remove the bottom 10 percent of our people. It isnt. Its
just the opposite. What I think is brutal and false kindness is keeping people
around who arent going to grow and prosper, argues the Chairman. Do you
agree with his views? Substantiate.

Answer
There have been instances in the history of corporations where the rank and yank
system, such as the one adopted in General Electric, has worked well. Jack Welch,
who was the Chairman and CEO of General Electric between 1981 and 2001 has been
one of the most vocal proponents of this system. He even gave a vitality model
described as a 20-70-10 system. According to this model the employees can be
divided into three categories: the top 20 per cent, the middle 70 per cent, and the
bottom 10 per cent. While the top 20 per cent of the workforce is most productive,
the vital 70 per cent work adequately. The bottom 10 per cent is non producers and
should be fired.
However, there are certain limitations and drawbacks in adopting this model and
indiscriminately firing the bottom 10 per cent of the so-called non producers. Firstly
such a kind of forced ranking undermines team work. It can encourage employees to
engage in destructive and wasteful game-playing designed to ensure that they get
credit, or others dont. It can also result in lower productivity, inequity and
skepticism, negative effects on employee engagement, reduced collaboration, and
damage to morale and mistrust in leadership.
Also, firing employees for performance would be quite a tough thing to do because
it has real human consequences, and because it is an admission of management
failure, particularly if the employee is someone the manager hired in the first place.
There may also arise certain kind of morale problems among the employees from
what can be referred to as forced ranking.
Further, theres also a methodological flaw in such a rank and yank system. Its
based on a classic bell curve, wherein a vast majority of people will invariably fall
somewhere in the middle. Even if one could identify the bottom 10 per cent, firing
them would just give us a different bottom 10 per cent. The moot point where does
this end?
Still further, a forced ranking can only work if there are clear objective performance
criteria already in place. This is because without clear performance standards and
expectations, the decision on who is in the bottom 10 per cent would be left up to
individual managers, whose judgments can either become or at least perceived as
political and unpredictable. Thus firing the bottom 10 per cent would seem to be a
quick fix solution to a situation where managers are not doing their jobs properly and
the problems that the organization may be facing. Just ranking people without a
specific idea of the basis for the ranking can hardly tell us anything about whether, or
how well people are meeting an organizations goals, and hence is best avoide

16. In a state in India, government officials are regularly denying poor, illiterate
workers, their statutory minimum wage under a government scheme ensuring a
minimum level of employment for them. The actual quantity of work done is
being routinely under measured. False entries in employment registers with
bogus names, is enabling project foremen to pocket the payments. Other
malpractices including inflated estimates for public works projects, use of poor
quality materials, and over billing by suppliers are being followed. What can be
done to resolve this problem? List the measures that can be taken to plug such
loopholes.
Answer
Measures that can be taken:

Develop and enforce transparency safeguards and a methodology of muster


roll verification: Some of the methods can be pro-active disclosure of muster
rolls, regular maintenance of job cards etc.

Few imaginative ways of muster roll maintenance can be taken up. For
example, each labourer has to enter her signature or thumbprint in the muster
roll every day by way of marking attendance. This ensures not only that the
muster roll is available for public scrutiny at the worksite, but also that large
number of people actually see it every day.

Different methods of payment can be set up. For example, paying through post
offices. This is an example of the separation of payment agencies from
implementing agencies,. This system virtually removes any incentive the
implementing agencies have to fudge muster rolls, since the payments are
beyond their reach.

Institutionalized social audits, involving routine verification of records


through participatory processes is one of the best method that can be utilized
to plug the loopholes. With social audit, transparency can be built-in the
scheme that can take care of poor quality material, over billing and inflated
estimates. This is because it is ultimately the user who is enforcing the
accountability.

17. You are posted as a District Collector in one of the districts in India. It has
been brought to your notice that a structure has been built by few members of a
religious community on the public land without getting due permissions. In

keeping with the guidelines of the Supreme Court of India, disallowing the
construction of any permanent religious structures on public land, you are
contemplating its removal. However, the leaders of the community in question
have requested you to permit the structure saying that it is for the period of
month long religious festival only. Moreover they say that there is no other
religious place nearby where community members can celebrate their festival.
Your seniors and the political leader of the area also support their views.
However you are skeptical that after the festival is over, it may not be easy to
remove the religious structure from the public land due to the involvement of
community members at large.
What will you do in such a situation?
Answer
1st Approach
Firstly, leaders of the community should be persuaded that though District
Administration fully respect the values and customs of their religion but due strict
guideline administrators have to suffer from penal action if structure is not removed.
Second, if leaders do not agree, then the matter should be discussed with the superiors
that are there any authority which can give such permission. If it is not possible, then
he should remove the structure.
If such solution is not available, then he should check whether it is possible to limit
the number of people attending the festival, based on law and order administration
available and with the help of leaders of the community? If it is possible, then he can
allow them to take up the festival because structure can be removed later if number of
people involved is not much. If it is not possible, then he should remove the structure.
2nd Approach
Community activities can be taken up on a government land but proper permission
needs to be taken so I will ask to that community to take requisite permission in
written manner.
As a district officer it is my responsibility to maintain law and order and peace in
district and also to ensure sentiments of community is not hurt. Therefore to maintain
such balance permission will be granted till festive month but after that demolition
will be carried out under municipal act, following proper procedures and discussions
with influential and important members of religious community such that peace
would not be disturbed.

18. Consider the case of a religiously orthodox couple in Wonderland. Their son
is dying of cancer and needs a bone marrow transplant. While the doctor is
devoted to reducing pain and preserving life, the parents refuse permission to let
the doctor give the necessary bone-marrow transplant. In their view faith alone
does the healing and moreover, permitting the treatment would endanger the
childs life because it would issue from lack of faith. The doctor is dismissal of
these ideas and puts off all treatment until the parents agree for the transplant.
While the clashes between these two worldviews continue, the child dies. Should
the parents now be tried by the law for criminal neglect? Was the doctor also
being stubborn and betraying weakness, or is he just being a doctor? Has he
violated his code of ethics too?
The principle of autonomy is one of the four guiding principles of medical ethics, the
others being beneficence, non-malfeasance, and justice. It means that patients have
the right to decide what is done to their own bodies. For minors, it is the parents
decide for them. What happens when parents refuse a treatment that their childs
doctor recommends?
While the faith of the parents should be respected, any possibility of a miraculous
cure based on their religious faith should be entirely left out of account. In the given
case, it was a near medical certainty that the child would die without treatment; and
the only effective treatment was probably a bone marrow transplant.
If the doctor believed that the parents were not acting in the childs best interest, he
should have gone to the court and tried to convince the judge that the court should
take temporary custody of the child and allow the treatment. Another way for the
doctor was to try persuading the parents to accept medical care. He could have even
discussed passages from the holy books of various religions (and probably even
theirs), which talk about healing and even prayed with them, as a sign that he also
respected their faith. He could have made them believe that he was not opposed to
their beliefs per se, and was not negating the power of their God, and in fact was a
part of it. In the process he could have tried to balance their view on faith and science
and told them to keep praying for their child, while he gives him the requisite
treatment. It was important that he tried to appreciate their point of view, rather than
dismissing it altogether.
In such a situation it was important to balance the practical obligations of providing
medical care while adhering to doctors ethical obligations to report such cases to the
police and the courts. To this limited extent the doctor violated his code of ethics and
abdicated his responsibilities. He was also being stubborn and betraying weakness.
The doctors decision on bone marrow transplant decision was made on the basis of
medical knowledge and experience, evidence and reason. Though by definition, a
faith based miracle defies medical science and all known experience and reason, it

was important that the parents be made to realize that they were simply engaging in
child neglect. Assuming that there are adequate laws on child neglect, the parents
should be tried and penalized in some way under the same.
However, it must also be kept in mind that even parents, who are blinded by religious
faith and miracles, love their children. They would have loved to have their child
survive, but they believed in healing with prayer. It is unlikely that they were harming
the interests of their child intentionally; its just that they had a strong faith that others
dont share.
The doctor should not have made a rash decision when faced with a family refusing
medical care, despite how he felt about the familys religious beliefs. If he would
have approached them in a more humanistic way and with compassion, compromise
was much more likely. Finding common ground with parents about religious views
would have helped in developing a compatible care plan and maybe saved the child. It
was important for the doctor to guard against his frustrations, which held him from
working with the family in saving the child.
Thus both the doctor as well as the parents has to share the blame for the childs
death. While the doctor violated his code of ethics and was being irresponsible, the
parents are guilty of child neglect. Both could be tried and probably penalized in
different ways.
19. You have performed a competent piece of consulting work for Kumar
Industries and the CEO of Kumar Industries invites you to his company's New
Year Eve party, where he hands you an envelope marked Happy New Year
and murmurs, To be opened at New Year. It is just a small token of
appreciation. You get back to your office and open the envelope: inside are
hundred crisp, new one thousand rupee notes. Your office mate, an old hand at
the game, says: Keep it. A few companies do it around here. He'll just ask you
for some information from time to time about other companies that use our
services-nothing like industrial espionage, you understand, just a general sense
of how things are going with rivals, what's most on their minds, that sort of
thing. It's not as if you're going to be asked to reveal patented secrets or the
contents of files marked confidential. Do you keep the money? Or do you risk
offending a client valued by your company by returning the money? Does the
fact that the money is direct, not negotiated by agent to whom you pay a sum of
money for the service, make a difference and if so, how?
Answer
A gift is something of value given without the expectation of return; a bribe is the
same thing given in the hope of influence or benefit. In this scenario I need to enquire
about my organizations policy on gifts and hospitality. I will notify my line manager

or another appropriate person if I suspect that I have been offered a gift or hospitality
with corrupt intent or relationship will be influenced. If so, this is a bribe. In this
scenario principle of integrity need to be apply where In line with the global
professional and ethical standards you should always act with integrity. This means
being open and transparent in the way you study and work. It also means being honest
with others and never trying to hide the facts or purposely deceive those to whom you
have a professional responsibility.
20. You are the director of a unit in a regulatory agency that is charged with
monitoring the use of potentially harmful commercial chemicals. Geeta, a junior
project manager under your supervision, is responsible for studying a broadspectrum insecticide that is used not only in agriculture by small food- grain
farmers and cotton farmers, but also in the livestock sector as an animal spray.
She has been assigned to determine whether this product should be removed
from the market. At a social event, Geeta met a man named Siddharth, who she
later learned was the Mumbai representative for the insecticide manufacturer.
After meeting Siddharth several times, she became rather fond of him and
wanted to pursue the relationship further. However, Geeta realized that their
professional roles created a potential conflict of interest for her, and she decided
to tell you about the situation. She intended to continue seeing Siddharth and
said she considered herself mature enough to maintain a separation between her
professional and private lives. Geeta insisted that her feelings for Siddharth
would not influence her judgment in any way; in fact she and Siddharth had
never even discussed the chemical in question. What would you do in such a
situation? While you evaluate the alternatives available to you, what are the
moral codes and maxims that come to your mind as reference points for arriving
at a decision?
Answer
In this case the ethical situation is not much clear. Has Geeta done anything that
represents a breach of professional ethics? Because of her relationship with Siddharth,
it might well be difficult for her to maintain objectivity in discharging her duties. But
perhaps it might not be. People differ in their ability to manage tensions of this kind.
And what is your responsibility? Is it more important to avoid even the appearance of
unethical conduct within your organization or to support an employees right to
freedom in her private life? Should Geeta be trusted until her behaviour demonstrates
otherwise? Examine your alternatives and arrive at a suitable conclusion.
21. Mr. X worked in the human resources department that was interviewing
applicants for a top job in a widget company. After reading many applications,
one stood out way ahead of the others. Then he realized that he knew the
applicant, Mr. Y. They had hung out together when they were teenagers. Mr. Y
had been a wild kid and once was arrested for shoplifting and the possession of

drugs. Mr. Y had completed mandatory counseling and, as far as Mr. X knew,
Mr. Y had straightened out his life and had done well during the last twenty
years. Mr. Y hadn't indicated in his application that he had once been arrested
and Mr. X feared that the company would never hire someone with a police
record, no matter how minor or how long ago the offence was. Do you favor Mr.
X revealing the information?
Answer
In this case, principle of integrity applies. In line with the global professional and
ethical standards you should always act with integrity. This means being open and
transparent in the way you work. It also means being honest with others and never
trying to hide the facts or purposely deceive those to whom you have a professional
responsibility. So Mr. X need to tell the company and let them decide what is the best
for company's interest. Moreover in above case Mr. Y has already committed a crime
by not disclosing all the information to company regarding his arrest or conviction so
this also need to be taken into account where it is reflected that above person in future
could distort the facts for his benefit.
22. Imagine that you have been recently appointed as the head of the accounts
department in a municipal corporation. Soon after you assumed your duties, you
discovered that a clerk in your department was falsifying the payroll account by
continuing to carry the names of some employees who have already been
terminated. When the clerk picked up the payroll, he would pull out those
cheques, endorse and cash them and keep the money. You have no difficulty in
recognizing that this clerk is not only involved in unethical conduct but is also
clearly violating the law. In this situation, the following two options are available
to you. Which amongst these two would you choose as your response? Give
logical arguments in the support of your answer.
Answer
I will proceed with the formal charges and prosecution because of the following
reasons.

Any organization cannot sustain without public trust. There may be short term
gain without it.

The clerk should get fair and free chance to defend himself. He might get
smaller punishment then loosing the job. Thus, organization will not loose an
employee.

My image in my subordinates will be strengthened that I believe in justice and


fair play. Thus my authority and leadership will be enhanced.

If I fire him quietly, he becomes a mean to achieve the others goals.

I cannot expect that any situation might arise where I do not get a fair chance
to defend myself.

23. X is an excellent student and has always been at the top of his class in your
College. In his senior year he got selected to MIT. In February, his Internal
Assessment is due. X did his research and was writing his paper, he was using
some web sites for some last minute help. He has already written much of the
paper, but he finds an essay on his topic that fits perfectly. He cuts and pastes
some part of the topic. You notice the change in the writing style and then check
the web for plagiarism. Should you give the student a zero and report the
situation to the guidance counselor and principal so that they may communicate
the infraction to MIT?
Answer
If we look at the problem then X was a brilliant student. As he has always been at top
in his class and achieved a remarkable eligibility for higher studies in MIT, we can
say he was very hardworking also. He completed his research but at last moment he
took someone elses work and passed them off as his own.
Now we have three options. First option is we should give X a zero and report the
situation to the guidance counsellor and the principal. As per rules he will then
communicate the infraction to MIT. Second option is, considering it is a special
situation, teacher can reduce the punishment. We should ask the student to rewrite the
paper and can deduct certain marks from his final grades. Third option is we can call a
meeting with certain higher officials for this matter. We can present this case in front
of them and let them know our opinion i.e. punishment should be in such a way that
students future would not get affected.
24. You are judging a high profile case of a rich businessman who is accused of
holding back money of numerous investors in the country. You are facing
tremendous pressure from politicians and an industrial lobby, asking you to be
lenient towards the businessman as he has created close to a million jobs in the
country and his bad reputation will be disastrous for the economy. The following
are some suggested options. Please evaluate the merits and demerits of each of
the options:
A. Ask the senior judge to transfer you from the case.

B. Ignore all the pressure tactics by various groups completely.


C. Increase the duration of the court hearing to soothe down the pressure.
D. Hold the case in abeyance and ask for an inquiry report from the regulatory
body.
E. Ask the media to stop reporting the case with such fanfare.
Also indicate (without necessarily restricting to the above options), what would
you advise, giving proper reasons.

As the options have been already provided to the students, it is expected out of them
to clearly spell out the reasons for supporting certain option. It should reflect their
competence to understand the situation and act with prudence. Every option has some
pros and cons. It is important to reflect the values cardinal to you while choosing an
option.
Answer:
On analysing the merits and demerits of the options provided in the question, we
understand that:
a) By asking the senior judge to transfer myself from the case would ensure that I
would not face public pressure, scrutiny from media and industrial lobby; It would
relieve me from the position of dilemma and I could look after other cases in better
manner.
But by doing so, I shall be escaping from my duty. It would lead to dereliction of
duty; besides if everyone facing similar situation desires similar steps, it would lead to
a situation of chaos where one will not be willing to take the duty assigned to him.
b) By ignoring the pressure tactics by various groups completely, we can focus on the
merits and demerits of the case in an objective manner. It will ensure neutrality in
decision making and help in setup a benchmark for further judgements. However,
saying so is easier than doing so. There will be relentless pressure from various
groups citing issues like it will harm the economy of the country, it
could lead to loss of employment for many people, it may cause safety issue for the
judge himself and his family may face little harassment too.
c) By increasing the duration of court hearing to soothe down the pressure, the court
can fetch more time for itself to analyse the case. Besides, it will provide enough time

to the aggrieved parties to strengthen their case both for and against the issue. Also,
with time pressure from people will reduce and it will help in taking the best possible
decision in the matter. But by taking such step, it would be a case of delaying justice
and it is firmly believed that justice delayed is justice denied. Also, there is no
certainty that pressure will not mount back at the later date of hearing. Along with it,
the professional integrity of the judge will be questionable and it could set wrong
precedent for future in the country which is already reeling under the burden of large
number of pending cases in the judiciary.
d) By holding the case in abeyance and asking for an enquiry report from a regulatory
body, the judge will get expert opinion on the matter which can help in taking an
objective and better decision. It will also serve to reduce public pressure. However, it
has its own costs. It would delay the judgement and may cause trouble in making the
judgement if the enquiry report provides a completely different overview. Also, there
could be comparisons drawn from the conclusion of the enquiry reports and the courts
judgment, if both dont match.
e) By asking the media to stop reporting the case with a fanfare, the pressure from
public can be subdued to a certain extent. Many parties who are not directly related to
the case will reduce the pressure on the court. But doing so, there could be concerns
of judiciary trying to tame down media's right for freedom of speech and expression
and may lead to rumours of some misdeeds behind the settings. Also, it would be
against the basic theme of transparency and accountability, which are the backbone of
any public office. Thus, it may reduce the public faith in the judicial system of the
country.
On analysing the possible options, the best step possible, which can be taken, is that
the judiciary should ask for setting up the enquiry committee in case, if the matter is
requiring some expert knowledge on some topics, under consideration. Based on the
reports and other facts available with the judge, proper judgement should be taken
based on the merits of the law without any pressure from any external source.
25. Lisha Gupta is the environmental compliance manager for a small plastics
manufacturing company. She is currently faced with a dilemma on whether or
not to spend money on a new technology that will reduce the level of a particular
toxin in the wastewater that flows out from the back of the factory into a lake.
The factory's emission levels are already within legal limits. However, Lisha
knows that environmental regulations for this particular toxin are lagging
behind scientific evidence. In fact, a scientist from the university had been
quoted in the newspaper recently, saying that if emission levels stayed at this
level, the fish in the lake and rivers in the area might soon have to be declared
unsafe for human consumption.

Further, if companies in the region don't engage in some self-regulation on this


issue, there is reason to fear that the government backed by public opinion
may force companies to begin using the new technology, and may also begin
requiring monthly emission level reports (which would be both expensive and
time consuming).
But the company's environmental compliance budget is tight. Asking for this
new technology to be installed would put Lisha's department over-budget, and
could jeopardize the company's ability to show a profit this year.
The following are some suggested options. Please evaluate the merits and
demerits of each of the options:
1. Lisha should focus on her companys financial performance and profits and
forget the environmental issue as the compliance budget is already tight.
2. As the emission levels of the company are within legal limits, Lisha should wait
for new governmental regulations first and only after that should think about
taking any action.
3. Lisha should proactively report the matter to the higher management and
ensure proper measures are taken but still profits of the company should receive
the utmost priority.
Also indicate (without necessarily restricting to the above options), what would
you advise, giving proper reasons.
Answer:
1. The only merit in this option is that company will be able to keep up its financial
performance and retain its profits.
But this option will represent short sightedness on the part of the company. First of
all, as is already mentioned above, the government may step in and make
environmental regulations which may be much harder to comply with resulting in
severe financial costs in terms of both men and material. Besides environmental
issues are complex and if the problem is not tackled in its initial stages, it may reach
the tipping point, assume gigantic proportions and may not be solvable later. So, the
decision may back fire on the financial front itself negating the above merit.
Secondly, if the people get affected due to the rising levels of toxin then a lot of ill
will would be generated about the company. There may be public protests which will
further hamper the interests of the company both in financial terms and loss of good
will of its customers.

Thirdly, this action by the company will represent abdication of its responsibilities
towards the society. A company is not a closed system and it depends on its
environment for certain inputs and also affects the environment through its outputs.
So, it is necessary that it realizes its responsibilities towards the larger society as well.
So, w.r.t. this option, the demerits clearly outweigh the merits. Hence it is not a good
decision.
2. In this option, the decision may be right legally but not ethically. The purpose of
laws is to regulate and guide peoples behavior in a direction which is socially
acceptable. But ultimately laws can never take care of all kind of possibilities and also
need to be regularly updated. Hence being legally correct does not always means
ethical conduct.
With the merits and demerits of this case being same as discussed above, the above
decision may stand on the legal front. But on ethical front it does not take care of the
larger society and hence it is not preferable.
3. In this case, the concern showed by the company towards its externalities is
appreciable. Reporting the matter to the higher management will bring the issue to the
forefront and promote healthy discussions on the topic. The company can start by
taking small steps which may not be much costlier and prepare the ground for
combating the problem before it assumes gigantic proportions. The company can start
consultations with the government or public which will highlight its concerns and
generate a lot of good will for the company.
Besides it will be only practical for the company to give its finances higher priority as
it affects the sustainability of the company. If finances get affected, it may shift the
focus of higher management completely towards the finances with the environment
issue getting completely sidelined during later stages when it will become more
dangerous and wanting. Being able to manage the profits would also send a signal that
prudent financial management and environmental concerns can coexist.
One way is that company can take certain steps under its Corporate Social
Responsibility. It will on one hand help in creating goodwill for the company and on
the other hand not adversely affect the companys finances.
Recommendation
The only other option left apart from the above mentioned ones is that company
devotes its complete attention to the environmental issue without thinking about
profits. But that decision may affect the sustainability of the company and may have
repercussions as discussed in the second last paragraph of option three.

So, I will prefer option three as it takes care of both the issues at hand and presents a
solution that is in line with both financial and ethical requirements of the company. It
shows pragmatism and far sightedness on the part of the company.
26. Sahu has heard from his manager that their organization will be downsizing;
it could be as little as 5 percent or as much as 30 percent. However, the
supervisor told Sahu that "we're all under strict orders to keep it quiet" so that
the agency's best employees will not seek other jobs.
Mohan (one of the finest professionals in Sahu's unit), upon hearing the
downsizing rumors, told Sahu that he was sure that he could get another job at a
new business if a reduction in force occurred. However, openings at the new
business will close soon. Mohan asked Sahu, "Will there be layoffs?" and
"Should I get another job now?"
a) What are the options available to Sahu?
b) Evaluate each of these options and highlight which option you would have
preferred, giving reasons for the same.
Answer:
The dilemma in the above case is between the loyalties of Sahu towards his seniors
and his responsibilities towards the welfare of his juniors or team mates. Various
options available to him are:
1. Inform Mohan about the downsizing: This course of action will be advisable on
the grounds that Rohan will be telling the truth and also following his responsibility
of taking care of his unit members. But at the same time the action will be strongly
against the professional ethics an employee should follow. It will be betraying the
trust of your seniors.
Besides Sahu himself does not have any concrete information. All he has heard is in
the nature of informal communication. The scale of downsizing is also not clear. May
be it is very less and as Mohan is one of the finest professional, he may not be in
danger at all.
So, this course of action is not advisable.
2. Tell Mohan that he does not have any information regarding layoffs: This
action will be completely in consonance with the professional ethics with Sahu
keeping the trust and loyalty of his superiors. But it will not be right on the part of
Sahu to keep his unit members completely in dark and giving them no time to look for

alternatives which may be necessary for their career continuity. Some like Mohan
may completely miss out their existing opportunities resulting in hardships.
So, this course of action, though professionally correct, is also not desirable as it may
cause a lot of trouble to Sahus unit members.
3. Talk to the senior management regarding keeping unit members informed:
Apart from the above two options, one thing Sahu can do is talk to his superior about
the necessity of keeping the employees informed about the current scenario. He
should try to persuade his superiors to take employees into confidence by telling them
that finances of the company are strained and austerity measures may be taken. But
none of this is final as of now and there is no need to worry. Besides this is the time
that company needs the best performance of all its employees so that they can get out
of current economic problems. Also that the company stands by its best employees
and nobody will be given any unfair treatment. If any drastic measures are necessary,
due time will be given to them to make alternative arrangements. Further those who
stand with the company in these tough times will be suitably rewarded.
This may lead to certain employees looking for alternate jobs but that will be in the
interest of company itself because it will help in identifying who are the most
committed ones which the company actually needs right now. More importantly it
will save the company from the ill will it may have to face if the layoffs are done
suddenly without any prior information. Sudden restructuring may also result in even
good employees looking for other opportunities as they will be suspicious of their
future prospects as well as uncertainties in the companys HR policy. Besides, less
talented but faithful and committed employees are always better than talented but less
committed ones, especially in the times of crisis.
Preferred solution
We will prefer the third option as it presents a solution by keeping everybody
informed about the ground realities and giving everybody a fair say in the entire
process. It ensures that nobody is kept in the dark and everybody can decide
according to their preferences. And based on the reasoning above, it may cause
certain problems in the short run, but in the long run it will be the correct decision.
27. Mr. Shivaji assigns a research problem in a high school calculus class. The
problem is quite difficult and will require collaborative time and effort of a team.
Mr. Shivaji divides the class into groups of four students, gives them
instructions, and tells them when the problem will be due.
Parameshs group has an initial meeting and decides to divide up the work and
then collaborate the information. Sarah, one of Parameshs group members,
offers to write a particular section of the paper based on some great information

she found on the Internet. The other members of the group, including Paramesh,
divide the remaining work and proceed with their respective research.
One week before the project is due, Paramesh finds out that Sarah has chosen to
copy and paste most of her paper from the Internet source. Paramesh picks up
on Sarahs plagiarism and knows it is wrong, but he needs an A on this
research problem in order to pass Calculus. Paramesh confronts Sarah and asks
her to redo her paper without plagiarizing, but Sarah claims she is too busy with
her other class work. She assures Paramesh that Mr. Shivaji will never discover
the plagiarism, and that if he does, she will take complete blame for it.
Paramesh finds himself in an extremely uncomfortable position. He feels partly
responsible for the plagiarism because he is a part of the same group and he
knows about it as well.
The following are some suggested options. Please evaluate the merits and
demerits of each of the options:
1. Paramesh should forget about plagiarism as it is a common practice and
ultimately he is not the one who did it.
2. Paramesh should highlight the issue to the whole group and then decide after
taking everybodys advice.
3. Paramesh should talk directly to Mr. Shivaji and leave it to him to take the
right course of action
4. Paramesh should himself redo the work of Sarah and then submit the paper.
Also indicate (without necessarily restricting to the above options), what would
you advise, giving proper reasons.
Answer
1. The only merit in this option is that Paramesh may be able to get an A on this
research problem and pass calculus. But that is also not certain because there is no
guarantee that Mr. Shivaji will not find out about plagiarism himself. And if he finds
out than, even if Veenna confesses to her mistake, he will punish the entire group.
On the other hand Paramesh accepting plagiarism as a common practice will be very
harmful in his future career. It may result in development of a habit for such unethical
things which may create bigger issues in his further career. Ignorance of these small
things at early stages of life tends to create a propensity for such acts in later stages of
life.

Also it will not be correct for him to let one of his classmates indulge in such
unethical practices. As a friend and classmate, it is his responsibility to stop her from
doing such things even if it involves some confrontation. It is necessary to stop her
from doing such things as early as possible.
So based on the above discussion, this option is not acceptable at all.
2. Telling everybody will be the right course of action because if plagiarism gets
caught, then everybody will have to face the consequences. Since task is assigned to
the group, everybody will be held responsible for the unethical conduct. It may
generate some ill will about Paramesh in Sarahs mind. But Paramesh being part of a
larger group has a greater responsibility to think about everybody.
Besides, bringing the issue into light will also be in the interests of Sarah as well.
Telling others may also put some extra pressure on Sarah. This peer pressure may
force her to accept her mistake and rectify it.
Overall it will only be fair to inform everybody about the issue so that nobody is kept
in the dark.
3. Telling Mr. Shivaji directly will save Paramesh from any humiliation later. It may
also result in him being forgiven for coming out with the truth. But it will also mean
Paramesh abdicating his responsibility towards the entire group for self-interest and
self-preservation. Paramesh should first talk about the issue with the group. If the
group fails to agree on a particular course of action or decides to hide the issue then
only Paramesh should talk to Mr. Shivaji directly.
4. First of all, doing all the work himself may not be feasible at all for Paramesh. As
mentioned in the case study, the problem is huge and needs collaborative effort.
Secondly it may be in short term interests of Sarah, but in the long term it may
become problematic for her. As discussed earlier, plagiarism may become a habit
resulting in grave consequences.
28. A terrorist group states that it has concealed a nuclear bomb in Delhi. The
authorities have captured the leader of the group. He says that he knows the
location of the bomb. He refuses to reveal the location. Torture is guaranteed to
produce the information needed to ensure the diffusal of the bomb.
a) Is it ethically acceptable for the authorities to have him tortured to find
out where the bomb is and thus save thousands of lives?
b) Suppose instead of catching the leader, the authorities have captured his
16 year old daughter. She is refusing to cooperate with the authorities. In

your opinion is it morally justified to have her tortured to acquire


necessary information and save thousands of lives?
Arguments in favour of torture
The principle of utilitarianism can applied to say that torture will be justified in this
case. Since, thousands of lives are at stake, it is necessary to exhaust all possible
options. Torture can produce the information which will help in saving the lives of the
people. Further, since the person who has been caught is behind this plot and is not
innocent, he shouldnt expect to be treated as any other citizen. He has committed a
crime against society; therefore torturing him cannot be termed as violation of his
rights.
Arguments against torture
According to deontological reasoning, consequences do not play any role in
determining the moral worth of an action. Lying is always an immoral act no matter
what the consequences may be. Similarly, according to this reasoning (propounded by
Kant) torture will always be immoral since human life (of the terrorist) is being
considered merely as means. The inviolable nature of human dignity belies any
justifications for torturing the ticking bomb terrorist.
The situation becomes more complex in the second scenario. It is hard to adhere to
the utilitarian conception in the second case. In the first case, since the terrorist has
committed a crime, one could justify torture. However, the girl has done no wrong
and it becomes difficult to justify torture in this case. She has had no role in the
activities of her father. Her only crime is that she is not cooperating with her
authorities. Further, being a juvenile she cannot be held totally accountable for her
actions.
Thus even though one might support torture in the first case, it is not possible to
adhere to the same position in the second case. The girl and her father are separate
beings. The girl is innocent whereas his father is not. This fact cannot be ignored
while examining the moral justification of torture.
29. You are in a supermarket with your younger brother. Your brother draws
your attention to a young child stealing food packets from the counter. He looks
needy and he is taking advantage of the crowded store which the owner fails to
manage in peak hours. The owner has a reputation of being a greedy and mean
person. What will you do and why?
The values involved in the above case are Honesty and Integrity vs. Empathy.
The answer should justify why you have weighed one over the other. Also consider
that this will be a lesson for your younger brother to remember.

This innocuous looking situation, is infact an intricate one. Not only does the young
needy boy have to be handled with utmost empathy, also the value of truth, honesty
and integrity be taught, not only to the boy but also to the younger brother. Inaction,
in this situation would be a huge disservice to both these young boys, as they would
be deprived of a very crucial lesson of life.
The best solution, in my opinion would be to instruct the younger brother to approach
the boy and tell him that what he did was wrong and should return the stolen packets,
else he may be caught and punished, if not today, then some time in future. It would
then be prudent to approach the boy myself and help him in whatever way I canbe
it a meal this time or any small help that I can possibly give him. Bringing the issue to
the notice of the shopkeeper may be dangerous, as known for his greediness and
meanness, he may try to punish the child that can scar the childs psyche for life.

30. During your election duty you caught hold of a vehicle full of cash. On
inquiry, it was revealed to you that the vehicle belongs to a very popular
politician of the region and even the exit polls are predicting his victory. The
politician calls you up to release the vehicle. You try to approach your senior to
seek advice but can't reach him.
a) What are the options available to you?
b) What course of action will you take?
Answer
This is an administrative challenge wherein the student is judged on choosing sense
of duty over political pressure. The answer must clearly reflect the students logic
and reason for choosing certain course of action after weighing it over its pros and
cons.
The election officer is under tremendous pressure from a popular politician who is
interfering in his duties. This situation depicts an administrative challenge where the
officer has to find a suitable way out to perform his actions without challenging the
politicians authority.
The following course of action is in front of him:
As the officers senior is not approachable the onus of decision making lies on the
officer itself.
a) Directly refusing the politician to accept his orders

As a subordinate it is the duty of the officer to obey his superiors in administrative


setup, but until the commands do not deviate him from his sense of duty and
consciousness. As the politician has asked him to perform an action away from his
line of duty, refusing him will show his uprightness and honesty. However, this
abrupt and rash behaviour can create ripples in the organization which can disrupt its
working at such crucial time.
b) Accepting politicians orders.
Blindly accepting politicians orders will show the officer in bad light in front of his
juniors as well as public. Not only has the officer to perform his duties honestly but
also has to make sure that the organizations public image should not be tarnished.
Also it is important to boost up the morale of the subordinates to perform the action
with clear conscious; otherwise it will encourage malpractices in the organization.
c) The best course of action would be to explain to the politician that the money
caught during the raid has been deposited in the government security and only a
written order can help him retrieve the money packets. Also reason with the politician
that it is very difficult to save oneself from media glare and any such act will tarnish
the image of the politician as well as the organization at large.
31. You are a young DM in a district, which is seeing rapid mushrooming of
private Drug de-addiction centres. The centres are known to function in an
unethical manner and their business model involves a specific consideration for
cases of relapse victims. Although these centres charge at high rates, they are
known to give families of addicts hope and institutionalisation of addicts has
ensured maintenance of law and order, not seen in neighbouring districts.
Analyse the following options for their positive and negative impact on the
various stakeholders of the society.
(a) Shutting down of the private rehab centres altogether
(b) Petitioning the higher authorities to deploy funds for opening up of
government centres
(c) Setting up a medical team to delve into the matter more deeply and acting
only after their report
(d) Letting the centres function in the same manner as earlier
Answer:

The drug de-addiction centres appear to be providing a respite to the family of the
addict and society at large is saved from the security threat that these addicts may
pose in form of thefts, robberies, etc. However, it is important to see that the addicts
are themselves victims who fall into the trap of addiction and need help from families
and society at large
Therefore, a blanket ban on the rehabilitation centres as suggested in option (a) will
do much harm than good. With families and relatives having nowhere to go and being
largely unaware, the society would be doomed to suffer from economic and
psychological stress due to burden of addicts and a situation of lawlessness can erupt.
The government centres, as suggested in option (b) is a welcome step as these centres
will provide a model for other private centres. Additionally, public opinion will be
created on improved methods practiced in government centres and they would
demand the same from private counterparts.
Inaction, by merely waiting for the medical report as given in option (c), can have
serious repercussions. This is indirectly encouraging the malpractices performed by
private centres, blinding ourselves to the massive challenges of poor treatment and
indirectly promoting the mal-practices as performed by the mushrooming private deaddiction centres.
Letting the centres run unhindered, as in option (d) would be running away from the
moral and occupational duty of the administrator
It is important to attack the root cause of the problem by providing awareness and
education about the hazards of the drug problem alongside encouraging creation of
government centres in the medium term. Additionally, strict norms of medical support
need to be instituted, both in government and private centres. It is equally important
to work in parallel towards eradicating the cause of the problem by deploying a
medical expert team.
32. Samatha had a job at the local mall as an employee at Play-Around. The
Play-Around has different Play equipment and a little kid section. Her job
included working the front desk, making sure that people paid as they entered,
also informing them of the rules of Play-Around, hosting birthday parties, and
watching the children.
One rainy Saturday when Play-Around was particularly crowded, a mother and
her mentally disabled son came to Play-Around. Play-Around does not
discriminate and all children are allowed on as long as they meet the height
requirements. However, adults are not allowed on the Plays because they are not
covered by Play-Arounds insurance. The mother was informed about the rules
verbally by Samatha as well as by the posted lists of rules.

This woman disregarded the rules and chose to follow her son onto the Plays.
This is dangerous and against the rules because there is the possibility that an
adult could slip and fall, and if either she or another child got hurt due to her
actions the insurance would not cover it. This woman believed that her son
needed special attention and constant supervision.
When it came to Samathas attention that the woman was on the inflatable,
Samatha told the mother that she would have to get off, however, her child could
stay. The woman responded by yelling at Samatha, telling her that her child had
every right to be on the inflatable as any other child. Samatha agreed and
reminded the woman that she had not told the child to get off but only the
mother herself. The woman then responded by saying that she had to stay with
her son in order to help him.
The following are some suggested options. Please evaluate the merits and
demerits of each of the options:
1. Samatha should refer the matter to the manager of the Play-Around.
2. Samatha should make it clear to the woman that either she should move out or
she has to force her son out as well.
3. Samatha should make an exception considering the special needs of the child
in question.
Also please indicate (without necessarily restricting to above options), what you
would like to advise, giving proper reasons.
Answer
1. The merit with this option is that this will be the right course of action from
procedural standpoint. Since she is not responsible for making or interpreting the rules
but only following them, informing the above authorities means following the right
protocol. Another merit with this option is that it will bring the entire issue in the eyes
of top management. This may result in framing of certain exceptions considering
special needs of disabled people resulting in long-term solution of the problem.
But the problem with this attitude is the general lack of awareness and affirmative
action regarding the differently abled or disabled people. The physical agony of these
differently abled people is further aggravated by the apathy of government and
society towards their needs. Hence some empathy and equity on the part of society is
necessary to remove this feeling of alienation among them. The same argument
applies to Samatha as well. So her treatment of differently abled people with the same
yardstick as for normal children is not ethically justified. Even the constitution allows

and emphasises affirmative action for the welfare of differently abled sections of
society.
2. The above arguments are applicable in this case as well. The only difference in this
case is that Samatha is being proactive about her duties and herself taking initiatives
so that rules of play-around are followed. While her earnest and sincerity regarding
her job is appreciable, but once again this shows her apathy regarding the special
needs of disabled people. Besides the possibility of a permanent solution if the case is
reported to higher management is also eliminated in this case.
3. The merit in making an exception is that it shows awareness, empathy and
compassion on the part of Samatha towards needs of disabled sections of society.
Such affirmative action will generate a lot of goodwill regarding the play-around in
the eyes of the mother of the disabled child. This will assure her that society
understands and is concerned about the special needs of her children.
But the demerit lies in the fact that first of all it will be against the rules of the playaround. However the bigger concern is that if some accident happens, then not only it
will be dangerous for the mother and her child but also result in Samatha losing her
job and being blamed for carelessness.
Advice: Aristotle has said treating unequals equally and equals unequally will
create further inequality. So, Samatha should display empathy and compassion
regarding the needs of the disabled children. Though Samatha should not compromise
on the safety of the children or the mother in any case, but she should try to help them
by may be guiding them towards less dangerous equipment or section where the child
can enjoy with her mother without any harm.
And Samatha should ensure that she highlights the matter in front of top management
so that a long-term solution regarding the needs of differently abled children is
devised.
33. Ravina recently completed her B.Com and was extremely excited to be hired
for her dream job working for a Public Sector Bank. During her initial days, she
began to notice that funds from grants were being mismanaged and
misallocated. Some of her co-workers were also using bank property materials,
including cars, for personal business.
However, Ravina was most shocked by the hiring practices she witnessed at the
office. Applicants to the jobs were supposed to take exams that were invigilated
by bank employees. Ravina began to notice that the invigilators were allowing
applicants to cheat on the tests because the applicants had already been chosen
for the job. Many of these pre-chosen applicants were friends of current
employees.

Ravina reported what she witnessed to Mahesh, the Branch Assistant Manager,
who was second-in- command to the Branch Manager. Mahesh told her, You
heard nothing, you saw nothing, and you say nothing. Ravina was absolutely
shocked; not only by the corruption, but that it was deliberately being swept
under the rug.
Ravina I was in a dilemma. She really needed the job to pay off loans, and she
loved the actual content of the work she was doing. She was also concerned that
it would look bad to leave her first job out in less than a year, as well as tarnish
future chances to work in a government organization. On the other hand, she felt
extremely uncomfortable in her work environment due to the culture of
corruption.
1. What are the options available to Ravina?
2.Evaluate each of these options and choose the option you would adopt ,giving
reasons.
Answer
Ravina is caught in a typical dilemma where she doesnt want to leave the job but
cannot see such a work environment embroiled in corruption too.One should point out
how overlooking this matter would mean a lot of loss to public exchequer and
inefficiency in the working of the bank. As Ravina was unsuccessful in her attempt to
bring out the matter to Mahesh, her approaching the Branch Manager to report the
issue seems the most appropriate option in such a situation.
Some of the options available to Ravina are:
a) She can ignore the matter and proceed in her usual way as this does not impact
her directly and any action taken would adversely impact her personally
b) She should report the matter to Branch manager and bring to light how she
was told to keep the matter under wraps.
c) She can resign from the job as taking either of the first steps would not leave
her satisfied and in a comfortable situation
d) Become a whistleblower. She can bring the matter to the notice of someone in
the vigilance department overseeing the functioning of the bank as it is public
money after all that is being squandered through such activities.
Option a) As per the rights/duty, fairness and common based approach it is Ravinas
duty to uphold the integrity of the banking system. The money that is being

squandered away is public money and therefore any such activity is hurting the public
interest in the long term. Additionally, the faulty hiring practices would greatly
impact the working of the bank and bring down its efficiency and effectiveness
further hurting the interests of the public and the society at large. It would be
unethical to ignore the matter for the dire consequences it might have on the
organization in future.
Even though ignoring the matter would save Ravina from the trouble of getting into
any controversy and help save her job but as discussed above it would not be healthy
in the long run for either the organization or even Ravina.
Option b) because of the following reasons: Virtue approach: integrity and honor are
two virtues that one should embody and help promote. One must ask myself what is
the highest state of character one can aspire to? One obviously recognizes the virtue
of honesty and merit. Likewise, one should push himself/herself to think about the
values one should live up to, those being integrity and promotion of merit in this
particular case. Therefore, the most appropriate option seems to reporting the matter
to the branch manager.
Option c) Resigning from the job does not seem a feasible option considering the fact
that Ravina has to pay off her loan and any such action would not show well on her
CV and have an adverse impact on her career. It would save Ravina the trouble of
getting into any kind of problem with regards to the conscience but it would put her in
a great trouble as she would be out of job and would put her and her familys
livelihood at stake. Morover, such an action would mean running away from the
situation and punishing oneself for the fault of others.
Option d) Even option D seems like a possible option but whistleblowing is a double
edged sword and should only be explored once all the possible options have been
exploited. It may be possible that if the matter is brought to branch managers
attention, he/she may take a strict action against those embroiled in such activities.
Proceeding to blow whistle on the matter without exploring this option may be taking
an action in haste and puts the reputation of the bank in a danger.
34. You have been appointed as a General Manager in a Public Sector
Undertaking, a new chemical plant for which is to be designed and then
constructed. Your responsibilities included hiring the staff for this new plant,
ensuring a safe design and then starting the plant up after construction.
During your previous experience you had noted that a new type of technology
could be used in the plant in place of conventional technology employed in public
sector plants till now. The new technology is less expensive and has more safety
features. You have convinced the project designer to employ this new technology.

This is a unique technology and only one manufacturer provides it. After this
technology has been specified by the designer and purchased, the salesman of
concerned company visits you and invites you to a lavish party in a foreign land
with air tickets. You have not known the salesman, Jayesh, prior to the visit. You
had also no direct purchasing responsibilities; you had just wanted the new
technology for better safety in the plant.
1. What are the options available to you?
2.Evaluate each of these options and choose the option you would adopt,giving
reasons.
This is a classic case of ethical dilemma involving gift taking. Any decision taken in
this regard should keep the interests of the organization and public above personal
interests. Options available in this situation are:
1. Accept the gift from the supplier as it hardly amounts to be a case of bribery
and the action was performed without any prior knowledge
2. Accepting the gift after bringing the notice to a senior in the organization
3. Refuse the gift from the supplier in a polite manner, clearly bringing out
reasons as how even though it may be legally fine but ethically wrong to
accept the gift
4. Refuse the gift and bring the matter to the notice of a senior in the
organization, demanding a blacklisting of the supplier for any future
transactions.
The first thing to notice about this case is that it is not a paradigm case of bribery. In
fact, it is not a case of bribery at all. We might define bribery as remuneration for the
performance of an act that is inconsistent with the work contract or the nature of the
work one has been hired to perform. I did not act contrary to my obligations to the
employer, and in fact I acted in accordance with my obligations. Furthermore, the gift,
was given after my recommendations concerning the valves and (we shall assume)
without any prior knowledge and expectation of the gift. But any such action of taking
gift may have a number of moral implications. Such an act may also be problematic
for the organization in the long run. Some of aspects including the arguments for and
against accepting the gift have been presented, students can use them as a hint to
arrive at their own answers.
Arguments against taking the gift:
1. The size of the gift is morally troubling.

2. Knowledge of the gift could influence others to buy from the supplier, even if his
products are not the most appropriate for them. This might operate as a kind of bribeahead-of-time for other people in my plant or in other plants, even if I had no idea I
would be offered a trip. They might say, "If we buy from this supplier, we can expect
a nice gift."
3. In morality, one of the important questions to ask is whether you would be willing
for others to do the same thing you did. If every salesman offered gifts to people who
bought--or recommended the purchase of--his products, and every purchaser accepted
the gifts, the practice would of course become universal. Our first reaction is to say
this would neutralize the influence of the gifts. You could expect a bribe from
somebody, no matter whose product you recommended. Thus I might have been
offered a nice trip by whatever valve salesman made the sale. But this begins to look
like extortion if not bribery: a salesman has to offer something to even have his
product considered. Furthermore, smaller companies might not be able to offer the
lavish gifts and so might not have their products considered. This would harm the
competitive process. Furthermore, the gifts would probably tend to get larger and
larger, as each salesman tried to top the other one. Thus, the general acceptance of the
practice would have undesirable consequences.
Arguments for taking the gift:
1. As already pointed out that my action cannot be an example of accepting a bribe in
the true sense of the term. In order to be a true bribe taker, I would have had to make
his decision because of suppliers offer. Since the trip was offered after my decision
and I did not know about the trip ahead of time, the trip could not be a bribe in the
true sense.
2. My company may stand to benefit from the personal relationship between me and
supplier It may make it easier to get replacement parts for the valves and to get other
types of service from suppliers company.
3. Professional life should have its "perks." Business and professional life involves a
lot of hard work. Foreign trips and similar amenities add spice to life that is important
in terms of job satisfaction and productivity.
35. You have been studying for the past two weeks for your final exam in the
college. Your grade in the class is much lower than it should be, and your father
has warned you to improve it or there will be consequences. So declining
invitations for parties, restricting you time with friends, and spending hours in
the library, you have done a lot to prepare for this exam.

On the exam day you sit next to a mutual friend of yours that lives on the same
floor in the hostel. Talking with him before the test begins, you notice that this
friend has hidden a cheat sheet at the top of his backpack.
Ordinarily, you wouldnt be concerned about it; however, the professor has
already announced that only limited students would pass the exam. Even if
everyone does really well, the professor will divide up the grades to make sure
theres a limited amount of As and Bs.
1. What are the options available to you?
2. Evaluate each of these options and choose the option you would adopt,
giving reasons.
Answer
One faces a tough situation of personal morals and values which go against cheating
and the other of losing a friend and being branded a complaint monger. Students
carefully need to take all the options into account giving due weightage to the fact that
inaction on ones part to report the cheating may not only jeopardize ones own
chances but also the chances of many other deserving candidates in the class.
Some of the options available are (list is not exhaustive):
a) Ignore the incidence and treat it as a one off, as my friendship is at stake
b) Approach my friend after the class and warn him/her about the action
c) Report the incidence to the teacher and not keep it to myself
The first option is invalidated as ignoring the incident will not only be against the
greatest good for the greatest number of people , as opined by utilitarian approach but
would also depict characters of cheating, and deceit which would not be in
consonance with either your own characters or the character of your friend in the long
run.
Further, ignoring the incident on your part, and not confronting your friend may lead
to a slippery slope, where it might only act as an invitation for further and heightened
crimes.
The second option might seem to be a viable approach, confronting your friend for his
act, but it goes against the principles of Right/duties, fairness and common good.
Being a member of the class, I have a responsibility to protect the integrity of the
grading system. By letting my friend cheat, I am allowing the greatest good to
diminish because the class average will be distorted as a result, affecting a lot of

students. Though it might hurt my friendship, I should at the very least, anonymously
report my friend in order to preserve and promote the greatest good.
The third option is the most viable. I would report it to the teacher and not keep it to
myself. This will not only help in exemplifying the virtuous characters of integrity
and honor but will also help in correcting the characters traits of my friend which
might help him in the long run.
I must ask myself what is the highest state of character I can aspire to? I obviously
recognize the virtue of discipline and handwork, as evidenced by my preparation
leading up to this test. Likewise, I should push myself to think about the values I
should live up to, those being integrity and honor in this particular case.
Further, Just because it is my friend who is cheating, doesnt mean that her friend is
exempt from immoral behaviour. If I knew a girl that I disliked was cheating, would I
be more motivated to report her? Since all people are seen as equal, I should not hold
my friend to a different standard.
I could either confront her friend directly, or warn her that I will be reporting her to
the professor. I can respect my friends dignity by providing her with transparent
options. Either way, I cannot allow the disparity in grades be perpetuated if I can help
it because the issue of passing the test is important to everyone and not just my friend.
They are all trying to obtain a passing grade, not just my friend. It is unfair and hence
unethical, for me to tip the balance in her favor.
36. Lisha and Ryan are two employees of Kishores organisation. One day the
two were talking in the hallway about the employee benefits program. Ryan, who
has had some recent financial trouble, explains to Lisha how the benefits
program has a loophole that will allow him to receive some financial assistance
that he really needs to help pay health care costs for his mother. Kishore
overhears the conversation. Later, Kishore is approached by her supervisor who
says he heard a rumor that some people were taking advantage of the company
benefits program.
1. What are the options available to Kishore?
2. Evaluate each of these options and choose the option, which you recommend
Kishore to adopt giving reasons.
Answer:
Options available to Kishore are:
1. Do not tell supervisor anything.

2. Disclose the entire incident to the supervisor.


3. Tell supervisor that she had also heard the some rumors and the loop holes in
the program but not tell him about the specific incident. Also confront Ryan
and tell him that higher management is already alert about the issue.
Evaluation of options:
1. Not telling anything will be a violation of professional integrity. This will make
Kishore also an accomplice in the ethically wrong act of using the loophole to derive
undue benefits. Besides Ryan also needs to be stopped from using such tactics as he
may develop a habit for such things resulting in bigger problems later. So, the option
is not advisable at all.
2. Disclosing the entire incident will be legally correct and in line with professional
and organisational ethics. But being a part of the organisation, Kishore has also some
responsibility towards her colleagues and their special needs. She should at one hand
try to help Ryan regarding her mothers condition and at the same time stop him from
using unethical tactics. Since disclosing the entire incident to the supervisor is bound
to result in severe condemnation of Ryan, this option is also not very desirable.
3. This option provides a good middle ground as it will bring the matter to the
knowledge of higher authorities and result in plugging of the loop holes so that the
incentives of its misuse are minimised. At the same time it will also stop Ryan from
resorting to any unethical practices in vulnerable circumstances.
Opinion: I will recommend option three as the right course of action for Kishore.
What I will further recommend is Kishore making a case before her supervisor
regarding the inability of current family benefits program of the company to satisfy its
employees needs. No program can be perfect and there will always be possibilities of
finding loop holes and misuse. The right course of solution is to devise the program in
such a way that it satisfies the needs of the employees and they are not forced to
search for loopholes. This will help in finding out a long term solution resulting in a
win-win situation for both the company and its employees.
37. You are a young reporter who has unearthed the case of the government
secretly pledging gold to a foreign country. Reporting of the said activity would
trigger a downward spiral for the economy as trust in the government would
decrease, domestic and foreign investors will pull out investments from the
country. The reporting of the incident would definitely improve your career
prospects. You are still to share this information with anybody.
What will you do and why?

Self regulation is the mechanism, most capable of striking the right balance between
freedom and restraint.
This is a case of an ethical dilemma between freedom of expression and censorship on
the part of the reporters self. In such a solution, a possible act can be:
Reporting the matter to the editor, suggesting it to be a cover story: This sounds
lucrative for someone who would be blinded by future career prospects without
giving due weight age to the serious repercussions that can follow.
The two sides that need to be considered judiciously before reaching a decision are:
Citizens right to know versus governments right to secrecy for betterment of the
country.
Apart from the personal gains of the reporter, it is more important to work towards the
larger goal of betterment of the society. Achieving the glory of breaking news that
leads to losses of investment in the country may not be satisfying and morally correct.
Also, it may not be easy to merely side with the government without questioning the
authenticity of the act. The possible solution therefore must be generating a wellinformed public debate and generating public opinion on the issue to understand the
gravity of the situation.
Above all, the reporter or the press at large is a bridge between government and
citizens. Taking sides for profit will ruin the spirit of press. Therefore, the decision
taken should reflect the good intentions of the reporter that he works for the
betterment of the society and not merely for personal glorification.
38. Namitha works in the Human Resource Department in a Marketing firm.
Marketing job entails a lot of travel, and accordingly the firm provides for many
perks. She finds that some employees engage in activities such as getting doctors
appointment during office hours, making personal phone calls, using office
computer and Internet for personal reasons. As a new employee, she finds this
behaviour unethical but her colleagues tell her that since the employees tend to
spend so much of their weekday hours on the job, it is all right to engage in some
personal work.
Consider the following options and evaluate them:
(a) Strict monitoring of the employees for such unethical behaviour.
(b) Ignoring this issue altogether.

(c) Dismissing the employees who are found to be engaged in such activities
on a regular basis.
(d) Reporting the matter to the Departmental head.
(e) Issuing warning to the employees.
Answer
The major issue in the above case is the issue of integrity and honesty in professional
life. Using office assets for personal gains and doing personal work during office time
means a loss of integrity and violation of professional ethics. This not only means
inefficiency in the working of organization, but promotes unhealthy work culture
which is detrimental to both the employee and the organization.
(a) Strict monitoring of the employees will ensure that they do not engage in
personal work during office time. But this might prove to be difficult or may
not be possible at all. Besides what needs to be understood is that monitoring
will only increase compliance. But the employees will involve in such
unprofessional acts again as soon as they get a chance. The real need is
inculcation of values of integrity and honesty in the employees. This cannot be
achieved by strict monitoring.
(b) Ignoring the issue will let status-quo prevail. While this may save the
company from the immediate backlash that may follow if stringent steps are
taken, but in the long term the effect will be very damaging. The problem may
reach such proportions that even the ethical employees will find it difficult to
carry on their work. The whole work culture will get corrupted with such
unethical practices becoming the norm
(c) Dismissing employees who regularly do this will generate some amount of
fear against unethical practices. This will create examples that violation of
professional ethics will not be tolerated. However such strict action should
also be supplemented by incentivizing ethical behavior. Also as noted above,
real change will come only through an inculcation of values and development
of an ethical work culture.
(d) This will make sure that those in policy making position are aware of the
unethical practices followed in the organization. This will help in bringing the
issue to the notice of authorities and corrective action can be taken. The higher
authorities should also not only try to stop these unethical practices, but also
try to remove the real reason behind them. As it is noted above that the work
involves a lot of travel, authorities should ensure that employees get enough

time to deal with their personal problems and issues. They should realize that
company policies are also to some extent responsible for the current state of
affairs and hence they should also take care of interests of their employees.
(e) Employees who tend to engage in personal work have the notion that such acts
entail no consequences. A strict warning will only be fair because the misuse
of office time has become an accepted part of work culture. Unless the
employees are made aware of the fact that the company has taken cognizance
of this issue, it cannot be expected that they will change their attitude.
The real need of the company is to promote a healthy work culture so that unethical
unprofessional conduct is minimized. A correct step in this regard will be the
company bringing out a code of ethics expected to be followed by its employees. This
should be supplemented by a code of conduct providing specific guidelines of what
kind of behavior is permitted and what is strictly prohibited. There should be proper
mechanism for enforcement of the code of conduct and punishment for violations.
This will help in laying to rest the claims that some personal use of office property is
justified. This coupled with a rationalization of employee job profile so that he gets
enough time for his personal needs will ensure the development of a healthy work
culture in the organization.
39. You are in charge of the rescue operations in an epidemic hit area,
commanding a team of 6 medical professionals. The area is located remotely and
not well connected with the nearby town through roads. It takes around 24 hours
to reach the nearest district hospital, which has the required vaccines and drugs.
Thousands of people have died because of the disease. The disease is contagious,
though it gets transmitted only through bodily fluids and not through air.
However, one of your team members has fallen sick and his condition is critical.
Moreover, the team does not have the required drugs to treat him. Your team
members request you to call off the operation as they are scared of the disease
and want to take back the sick team-member to the hospital. At the same time,
media has been portraying your efforts in a negative light.
(a) What are the options available to you?
(b) Evaluate each option and suggest the best course of action?

Answer
The following are the options available:

1. Call-off the rescue operation and take back the affected team member back for
the treatment.
2. Continue the operation and try to keep the sick team members condition
stable
3. Report the situation clearly and assistance required to higher authorities. Ask
the government for air-lift.
4. Send off a few members of your team to take the affected member back and
the rest can continue the rescue work.

Calling off the operation will be an over-reaction and abandonment of the public
service value. Even though the disease is contagious, it gets transmitted only
through bodily fluids and not through air. Hence, there is no need to press the panic
button. Necessary precautions might be taken to keep the risk of contagion at bay.
Moreover, such an act will definitely not go down well with the affected people,
administration or the media which is already perceiving efforts in negative light. This
will further affect the rescue operations as support of the public and media is very
important in ensuring the success of the operations.
The positive associated with such an action is that the precious life of the team
member would be saved.
Opting for the second option may not be a prudent thing to do either, given that the
team does not have the required drugs to treat him. Doing so will reflect ones
disregard for life and to the commitment of the team members. It will adversely affect
the confidence of the team-members in the leader, affecting the overall team morale
Moreover, keeping in mind that the disease is contagious, instead of keeping the
affected persons with oneself, the focus should be on quickly moving the affected
people by transferring them to places like district hospitals where they can not only be
treated well but also stopped from spreading the disease further.
The situation should clearly be reported to the higher authorities and assistance may
be demanded. Though the government may be asked to provide for an airlift but it
will depend on the resources available with the administration and the weather
conditions at that point of time. Generally in a crisis situation the government
resources are spread out so thin that some delay is bound to happen. One has to make
a decision whether the condition of the affected team member permits that time
window. There may be a lot of variables involved.

As one has a team of 6 at ones disposal, it will be more prudent to send a couple of
team-mates along with the affected team-member to the nearest district hospital.
Local help, if available, may be taken too. Also, the other affected patients may also
be sent along to provide them with quality treatment and quarantine them. Moreover,
the nearest district hospital, which has the required vaccines/drugs may be able to
provide the team with the necessary drugs, water and food items on their way back. It
will also help the administration to have a clear idea of the situation at ground.
One has to showcase leadership to lead the team and community as a whole out of
crisis, motivate them and make use of networking/coordination skill to cover all
aspects of rescue, order, reporting as well as deciding on critical instances.
40. A deadly epidemic has broken out in West Africa killing thousands of people
in a short span of time and infecting many others. There is no vaccine or
treatment available to cure the disease. You are the head of the health
department of one of the countries that have been severely hit by the epidemic.
While there is no treatment currently available for the disease, an experimental
drug is being used in some other country. This drug has shown positive results
and consequently some of the patients have also managed to recover from the
disease. However, it is not yet clear whether the relief came due to the drug
alone, or otherwise.
Given the high mortality rate of the disease and its rapid spread, other countries
affected by this disease have already given a go-ahead for the mass production of
this experimental drug. But some health experts are skeptical about the efficacy
of the drug and have even warned against its use as it may result in some deadly
side-effects.
(a) Bring out and discuss the ethical issues involved in the above
case.
(b) What would you do in the above situation? Give reasons.
Answer
The answer should consists of the following parts
1) The facts of the case
2) The major ethical issues involved
3) Different options available to you and their analysis
4) The course of action you would adopt, giving reasons

The facts of the case are


The epidemic is deadly with a high fatality rate
There is no cure for the virus except for an experimental drug whose efficacy is
questionable
Some countries affected by this disease have given a go ahead for use of the
experimental drug.
The ethical issues involved here are
The disease is causing havoc among the population. As a head of Health department it
is my duty to take steps to tackle the situation. The experimental drug has not proved
its efficacy in tackling the disease. Without proper clinical trials it would be unethical
to give this drug to the victims. Also, some health experts have warned against the
possible side-effects of the drug. Thus administering the drug may prove to be worse
than the disease.
But in the absence of any other medication and the rapid spread of the disease, means
that some urgent action needs to be taken before the situation goes out of control.
Time is not a luxury that I have. Thus the question comes down to whether to go
ahead with the current cure or wait for clinical approval of the drug.
Options available
1) Wait for clinically approved drug This option will save me from running the risk
of any side-effects that may arise from using the experimental drug. But as the
epidemic is spreading rapidly and finding a sure-fire cure may take time, it may be
possible that a large population will be affected with a huge death-toll before the cure
is administered to the people. Hence, this option is not suitable.
2) Give go ahead for the use of the experimental drug The situation is very dire in
my country and it requires immediate action. This will ensure that some sort of action
is taken on my part. But it is also possible that the cure may prove ineffective and in
worse case scenario, affect the population adversely. Hence, without any clinical
approval, it is not suitable to administer the drug to the population.
3) Wait to see what impact the drug has on the neighboring countries affected
population and act accordingly This option accords equal importance to gauging the
efficacy of the drug and also keeping public health in mind. If the drug shows positive
signs in other countries and no side-effects, then I will also give a go ahead to use of
this drug. But if the drug proves ineffective or results in major side-effects than I will

not allow the use of this experimental drug. This will save the population from
something which is worse than the cure, though the delay may cause some additional
deaths.
I will choose the 3rd option as it is an informed decision. Though the wait period may
result in some deaths but the larger affected population will be saved from being
administered a wrong cure. This option shows that instead of panicking under the
outbreak of epidemic and going with any available untested cure, I have followed an
informed course of action.
41. You are the District Magistrate of one of the poorest districts in the country.
Due to widespread poverty and unhygienic conditions, there are many health
related problems in the district. Many private medical practitioners have settled
in the district and are running a profitable business. There are reports that
many of them are charging high fees even for routine medical check-ups.
Further, out- of pocket health expenditure has risen exponentially in the district.
The MP of that constituency has fixed the rates, which the doctors can charge
from their patients, for the most common medical treatment. These rates are
within the capacity of the poor masses and would not impact the earning of the
doctors too. He has also warned that if any doctor is found to be in violation of
these, then serious action would be initiated against him.
The medical fraternity has protested against these arm-twisting tactics. They
have approached the DM to remedy the situation.
(a) What are the options available to you?
(b) Evaluate each option and suggest the best course of action?
Answer
The facts of the case are
1. The public health condition of the district is poor
2. Some doctors are charging very high fees
3. The MP has fixed the rates which the doctors can charge from their patients,
for the most common medical treatment.
4. These rates are within the capacity of the poor masses and would not impact
the earning of the doctors too

5. The medical community is against the rate list fixed by the MP


The options available to me are
1. Ask the doctors to follow the rate list As the public health of the district is in
abysmal state, hence limiting the medical charges is a good option. The
Minister of Parliament is an elected representative and his orders should be
respected. But the unilateral declaration made by the MP is not acceptable.
Such diktats are signs of a dictatorship and not a democracy. Hence, this
course of action is not suitable.
2. Ask for the MP to revoke his diktat The MP has used extra-constitutional
means to address the poor health situation of the district. Though the intention
behind the act is noble but the method used is not appropriate. But asking the
MP to revoke his diktat is only a partial solution as it doesnt address the high
charges charged by some doctors for routine medical check-ups. Hence, this
option is also not suitable.
3. Refuse to intervene in the matter By refusing to intervene in the situation, I
am saved from the ire of the Minister. It may prove beneficial for my career in
the future. But I am abdicating my responsibility as a District Magistrate. This
shows lack of conviction on my part. Also, the doctors may go on strike or
approach the Courts for redressal for their grievance which will be a bigger
headache for me. Hence, this is also not a suitable option.
4. Arrange for a meeting between the MP and representatives of the medical
community to resolve the matter Here, I have followed the middle path by
addressing the grievance of the medical community as well as taking care of
the concerns of the Minister regarding the poor public health of the district. I
will try to negotiate a solution whereby the freedom of profession of the
doctors is protected and also advice the representatives of the medical
community to guard against charging high fees. If still the MP doesnt agree to
revoke his diktat than I will assure the doctors that no harm will come their
way. Also, if the unethical practice of charging high fees is not curbed then I
will take strict action against errant doctors.
I would follow the 4th option. This is a pragmatic approach. I will ensure that both the
doctors as well as the MP have their way. The doctors will be able to practice their
profession freely and also the practice of charging high fees will be curbed which will
satisfy the Minister.
Also, I will urge the MP to provide funds from his MPLAD scheme for opening up of
new hospitals and improving CHCs, PHCs under NRHM to improve the public health
situation.

42. You are the Station House Officer of a police station. Recently, there has
been a spurt in theft incidents in your area. Some of these incidents have
occurred even during daytime. The Resident Welfare Association has blamed the
labourers, residing in night shelters, for the crimes.
The night shelters are a result of an initiative by the police to help the homeless
people to seek refuge from the biting cold at night. You have taken great pains to
get these night-shelters up and running. The record of all those who stay in these
night shelters, for instance, their names and identity proofs, is kept with the
police. But the police has no information if any one of them has a past criminal
record.
Even as an investigation regarding the thefts is going on, the RWA members are
demanding to dismantle these night shelters and to not allow these labourers to
reside near their homes.
(a) What are the options available to you?
(b) Evaluate each option and suggest the best course of action?
Answer
The facts of the case are

The night shelters help the poor to escape from the biting cold in the night

There have been many cases of thefts in the locality in recent times

The police dont have the past criminal records of the residents of night
shelters, if any.

The options available to me are


1. Dismantle the night shelters Dismantling of the night shelters will address
the concerns of the residents living in the locality. It will save one the
headache of having to deal with the protests by these residents which may
escalated if the night shelters are not dismantled. But dismantling the shelters
without doing a thorough inquiry is not appropriate. Also without the shelters,
the poor labourers will have no place to stay in this biting cold. This option
shows a biased and elitist approach on the part of the administration. Hence, it
is not suitable.

2. Ignore the concerns of the residents of that locality and carry on with the
investigation As there is no proof that the labourers residing in these shelters
are responsible for the thefts, hence there is a need for thorough investigation.
But ignoring the concerns of the residents altogether reflects on insensitivity
on the part of police. The residents may escalate the matter further by
approaching the higher authorities which may generate extra pressure on the
SHO. Additionally, a as a duty bound officer it is the responsibility of the
SHO to provide a safe living environment to the residents of the area. Hence,
this option is also not entirely suitable.
3. Shift the night shelters to some other location Shifting the night shelters will
appease the residents of that locality and it will generate goodwill for the SHO
among the people. But without any inquiry and merely on the complaint of the
residents, shifting the night shelters in not a correct approach. Also, since it is
an initiative of the police to set up the night shelters, shifting them may not be
possible because of paucity of funds etc. In the meantime, there will be
inconvenience caused to the labourers. This problem doesnt address the root
cause and hence is not suitable.
4. Expedite the investigation process while ensuring a strict vigil in the area for
the time being This option ensures that no inconvenience is caused to the
labourers and also the concerns of the residents are addressed. I will do a
thorough background check of the people staying in the night shelters and also
expedite the investigation process. In the meantime, I will increase
surveillance in the locality to ensure that no further thefts occur in the area.
I would follow the 4th option. It shows that I have not bowed down to unreasonable
public pressure and taken an unbiased and fair decision. I have also ensured that no
future thefts occur in the area thus assuaging the fears of the residents. Also, I have
ensured that the labourers are not unnecessarily harassed and that they have a place to
stay in the biting cold.
43. You are a final-year engineering student, studying in a reputed engineering
college of India. Campus placements are going on in your college and most of
your friends have already been placed in reputed multi-national companies.
You have appeared in selection tests for many companies but have failed to clear
any of them. The placement season is drawing to a close. You have taken a
student loan to pay for the college tuition fees. Your parents have high hopes
from you and placement in a good MNC is very important for your future.
Some of your friends have offered to help you to cheat in the next selection test.
Many of them have themselves cleared their tests by cheating and now have job

offers from big MNCs. You have never cheated in any exam in your whole
student life. But the present situation is making you indecisive.
(a) What are the options available to you?
(b) Evaluate each option and suggest the best course of action ?
The facts of the case are
1. I have failed to get selected in any company
2. The placement season is drawing towards its end
3. Cheating in the selection test may help in getting me a job
4. I must get a job to take care of my financial obligations
1. Resort to cheating As I have to pay-off my student loan, hence getting a job
is very important for me. This will ensure that I will get a good job and my
future will be financially secure. But the unethical means dont justify the
ends. Following unethical means to start my career means that in the future
also I may resort to such unethical acts to further my career. The fact that my
peers are resorting to such acts doesnt justify the fact that I too resort to
cheating. Such unethical acts which are not checked in their infancy come to
haunt you later. Thus, it will only lead me to a path of destruction. Hence, this
option is not suitable
2. Do not cheat As I have never resorted to the unethical means of cheating
ever in my life, hence starting now is not a good step. Also, cheating may
solve the immediate problem of me not getting a job but it doesnt address the
root cause as to why am I being not selected by the MNCs. But by following
this option, there is a chance that I may not get a job at all. This will be
disastrous for my future as I have many financial obligations. Without a job
my future will be insecure. Hence, merely not resorting to cheating is only a
partial solution.
3. Approach the placement cell for guidance Clearly, I have some issues due to
which I am not being selected by different companies. Cheating will not
resolve this root cause. Also, not cheating, while an ethical approach isnt a
solution to my predicament. I need help and guidance to correct my
shortcomings which could help me succeed in getting placed in a reputed
MNC.

I will follow the 3rd option. It may be possible that I may not get selected in any
MNC but atleast I will emerge from the situation with an ethically upright character.
It is said that if wealth is lost nothing is lost but if character is lost everything is lost.
Hence, loss of character can never be recovered.
I will also try for off-campus placement so that I leave no stone unturned. Also I will
inform the placement cell about the unethical practices used by some students to get
placements. This will show that not only I am honest but I also have integrity.

Potrebbero piacerti anche